CR Assumption - Solutions To 85 Questions PDF

Download as pdf or txt
Download as pdf or txt
You are on page 1of 60

Assumptions Latest Questions

1. We know the following:


a)Parents are reluctant to subject children to the pain of injections
b)adults, who are at risk of serious complications from influenza, are commonly vaccinated

c)A new influenza vaccine, administered painlessly in a nasal spray, is effective for children

From the above the author concludes that since nasal spray can be used effectively only on children and
since children do not develop any serious complications from influenza, the widespread use of nasal
spray vaccine does not have any serious health benefit.

This conclusion will hold true ONLY IF we can be sure that adults do not contract influenza from children.
Only then will there be no significant health benefit in using the nasal spray.

A. Irrelevant. We know that this vaccine is effective only on children. Therefore even if this weren't true, it
would not make any difference to the conclusion.

B. Irrelevant. Once again, assumptions about the working of the vaccine is irrelevant to the conclusion.

C. Irrelevant. We are discussing the benefits of using nasal spray vaccine. The price of the injectable
vaccine does not concern us.

D. CORRECT. Let us try the negation of this. Adults contract influenza primarily from children than have
influenza. IF the negation were true,then the conclusion would become incorrect as there would then be a
huge benefit to suing nasal sprays for kids.

E. We already know this to be true in the passage. “A new influenza vaccine, administered painlessly in a
nasal spray, is effective for children.” Therefore this becomes a fact and not an assumption.

2.
Goal This Year: To reduce the number of truckloads of refuse(garbage) to be incinerated to half of last
year's number.

Which of the options are essential to achieve this goal?

Before we begin, let us say 1 ton of ash was generated per truckload of refuse(Garbage) last year.

A. This does not matter. As long sufficient refuse has been separated for recycling to ensure the amount
of ash generated is half of last year, it does not matter if some materials that could be recycled are incin-
erated.

B. The cost of the process is irrelevant to the conclusion.

C. Please note, we only need to seperate enough refuse for recycling to reduce the number of truckloads
of refuse to be incinerated to half of last year's number. So if 60% percent of the refuse collected last year
was recyclable and only 55% of the refuse collected this year can be recycled, our goal can still be
reached.
D. CORRECT. The amount of refuse that will be recycled is being decided based on the amount of ash
generated last year. Once the separation has been made, if the non-recyclable refuse produces 2 tons of
ash per truckload(say) then our goal will not be met despite recycling
last year 1 truck load of residue generated 1 ton of ash. This year (after u pick out recyclable waste), 1
truck-load generates 2 or 3 or even more tons of residue ash, then the city will not achieve its objective.

E. As long as we are able to separate out enough refuse to recycle it, this will not affect the goal. It is
possible to have waste in excess of last year and still achieve our goal.

3.

Governor's Goal: to reduce crime rate


Governor's Plan: be harsher with the inmates by denying them access to college-level courses.
Argument Against the Plan: the people who took college level courses in prison committed fewer crimes
once they were released from prison. So Governor's plan will backfire.

Assumption : it is clear that the argument assumes the prisoners committed fewer crimes BECAUSE they
took college level courses in prison. This argument would crumble if we were told that the prisoners who
are less likely to commit crimes are the ones who opt for the college-level courses.

A. Incorrect. If this were true, then the governor's plan for removing college-level courses would not have
met with any resistance.

B. Irrelevant .The argument discusses whether inmates who have taken the courses are less likely to
commit a crime than inmates who have not taken these courses. The comparison is not with the general
population.

C. CORRECT. Let us negate this.


“ The group of inmates who chose to take college-level courses are already less likely than other inmates
to commit crimes after being released”
If the above is true, then implementing the governor's plan will not have any counter effect,as the prison-
ers taking up the college level courses that are being scrapped are already less likely to commit crimes.
Therefore the necessary assumption we need to make for the argument to hold is that all prisoners have
an equal tendency to commit a crime before they take on the college level courses.

D. Irrelevant, We are not discussing High school courses here.

E. Irrelevant to the argument.

4.

Conclusion- the current average height of Tufe’s Turfil sunflowers is undoubtedly at least partially attribut-
able to changes in Tufe’s environmental conditions

This means that the changes in the newly formed island's climate caused the sunflower plants to get
shorter.
Assumption: Here we are assuming the sunflowers on Tufe's island are affected in comparison with the
sunflowers on the mainland(kept as a standard for comparison).

What if the sunflowers on the mainland are in fact different from the original species (because of the
weather conditions on the mainland) while the sunflowers on Tufe have grown to the normal height( and
therefore are unaffected by the climate on Tufe island)?

A. Irrelevant. We are not discussing if the weather is beneficial. We want to know if it affects the natural
growth of Turfil sunflowers.

B. Irrelevant. Once again, we are wondering about the height of sunflowers and not about the numbers.

C. CORRECT. Let us negate this.

“The mainland’s environment has changed in ways that have resulted in Turfil sunflowers on the mainland
growing to be 40 centimeters taller than they did two centuries ago”.
This means the climate on the mainland changed too. How will the conclusion or evidence hold true in
that case? Mainland could have become wetter which made the plants on the main island grow more and
not the ones on the island grow less.

D. Irrelevant. We want to know if the weather conditions were “partially” responsible or not.

E. Irrelevant. This does not tell as anything new. Even if this were the only difference,this option neither
strengthens nor weakens the conclusion.

5.
this is a good problem to which to apply the REVERSAL METHOD for assumptions:

* REVERSE the assumption

* the argument should be DESTROYED

the argument assumes that not as many televisions are being made domestically in borodia. (this is why
they will have to import more.)

if we reverse (c), then the assemblers can put together TV's faster than before.

this means that FEWER of them will be able to make the SAME TOTAL NUMBER of televisions.

this severs the connection between the drop in the number of assemblers and the drop in the number of
televisions produced, so the argument falls apart.

(c) is CORRECT.

6.
Fact 1: the quantity of rice produced per year is currently just large enough to satisfy domestic demand.
Fact 2. Teruvia’s total rice acreage will not be expanded in the foreseeable future nor will rice yields per
acre increase appreciably
Fact 3: Teruvia’s population, however, will be increasing significantly for years to come

Conclusion: Teruvia will soon have to begin importing rice.

Now since the production of rice is not going to increase and the population will continue to rise, it only
makes sense that Teruvia will soon run out of rice UNLESS as the population increases the people begin
to consume less rice.

A. CORRECT. Negate this

“There is a pronounced trend of decreasing per capita demand for rice in Teruvia”
this would automatically destroy our conclusion and is therefore the assumption on which the argument
depends.

B. Irrelevant. Since this is already the case, the amount of rice produced wont decrease and hence wont
affect out conclusion.

C. Irrelevant. We know that the yield will remain constant.

D. We are concerned with the general population growth and not region wise growth.

E. Irrelevant.

7.
Conclusion: If these trends in fuel production and usage continue, therefore,

Sidurian reliance on foreign sources for fuel should decline soon.

Assumption: the only way the above conclusion can be true if is the rate of fuel consumption is either less
than or equal to the rate of fuel production in Siduria. If the fuel production increased as stated in the pas-
sage but the rate of consumption were to exceed the rate of production, Siduria will still have to import
fuel.

The only option that addresses this is A. A is CORRECT.

8.
Conclusion: In this case, therefore, since none of the seven small companies can afford to convert their
production lines to a new set of manufacturing specifications, only the three large companies will be able
to remain in business.

Assumption: We are told clearly that none of the small companies can afford the cost of conversion and
therefore WILL have to go out of business. This is a fact and cannot be disputed. Is there any other way
these small companies could stay in business? What if either one or all the seven small companies were
already following the standards the government plans to implement?

A. Irrelevant. If conclusion discusses the state of the companies once the products are regulated. What
will happen to the large/small companies if the products are not regulated will not affect the conclusion in
anyway.

B. Irrelevant. We already know the small companies are unable to absorb the cost of conversion. Wheth-
er this cost is greater for smaller companies is irrelevant to our conclusion.

C. Irrelevant.

D. We are discussing costs and not complexity.

E. CORRECT. Let us negate this:


All/few of the seven small companies currently manufactures the product to a set of specifications that
would match those the government would institute if the industry were to be regulated.
This means that for all/few companies there will be zero cost of conversion and they can continue to stay
in business. Without the assumption that none of the smaller companies currently follow government
specification,the conclusion will not hold good.

9.
A. CORRECT. Let us negate the option
Let us assume that every time a job has to be reworked, the company sends in more reliable and comep-
tent mechanics. Then the reason that rework is successful is because of the increased skill of the me-
chanic rather than complex nature of the job.

B. Irrelevant. We are discussing if complex jobs require rework due to the nature of the job or other exter-
nal factors. This option is irrelevant to the conclusion.

C. We already know this statement to be untrue. Passage states:


“Ace Repairs have to be reworked under the company’s warranty”

D. The negation of this option would help negate the conclusion ONLY if the passage stated that the
same mechanics who were penalised were sent for the rework. Since we do not know this to be true, this
option is incorrect.

E. Irrelevant We are only discussing complex jobs that have required repair. Even though there exists a
category of jobs which carries out first time jobs satisfactorily, it does not necessarily negate the fact that
the other jobs failures needs concentration and requires rework.

10.
Conclusion- Derderia will not break its record for the new jobs created unless a record number of compa-
nies start up this year

Let us consider a situation where the above conclusion will not hold water:
let's say no. of companies started last year: 10

No. of jobs rolled out/company: 5

Total jobs last year: 50

No. of comapnies stared this year: 6 (= less than 10)

No. of jobs rolled out/company: 20

Total jobs this year: 120 (= way more than 50)

looks like we've broken the jobs record, without breaking the record for the number of companies started.
so we don't need to have a record number of startup companies.

A. New startups may create more jobs. But this does not answer why the startups need to be more than
the number last year. What if a lesser number of startups began but added more jobs per startup than last
year?

Also here is what negation of A tells us:

start up companies do not create more jobs than established companies.

Even if the negation were true, the conclusion is STILL valid.

B. choice (b) deals only with companies established last year, whereas the stated premise deals with
companies established at any time before this year. the companies dealt with in choice (b), then, are only
a small subset of the companies dealt with in the stated premise.

however, we don't need to make the assumption in choice (b), as it's irrelevant one way or the other: the
companies founded last year are merely a subset of "previously established companies". therefore, since
the stated premise already gives us the overall result for ALL previously founded companies, we have no
need to be concerned with the results of smaller components of that population (such as (b)).

C. CORRECT. Fits the above scenario perfectly

D. it's dead out-of-scope. Any consideration of jobs lost is irrelevant.

E. Same as D :)

11.
A. Irrelevant. Clearly we are not concerned with the conditions that existed while the paintings were cre-
ated but rather with the conditions required to save those paintings.

B. Weakens,This weakens the conclusion slightly and does not provide any information about what would
happen the paintings were the museums to relax their standards.

C. Irrelevant. The passage clearly discusses the cost involved in storing renaissance paintings alone and
not other objects.

D. CORRECT. Let us negate this:

Other materials in Renaissance oil paintings other than the paint are vulnerable enough to relatively wide
fluctuations in temperature and humidity to cause damage to the paintings.

This would destroy the conclusion that “museums could relax their standards and save money without
endangering their Renaissance oil paintings” as it states that despite oil paint not being affected by tem-
perature fluctuations (like the study proves) other materials in the painting maybe.

E. Irrelevant.

12.

A. CORRECT. Conclusion states that “Clearly, therefore, insurance companies are making a greater prof-
it on collision-damage insurance in Greatport than in Fairmont” If it costs more at Greatport to repair
damages than it does at fairmont,then despite there being a lower number of collisions at Greatport,the
companies may not make greater profit.

B. Let us consider the negation of this:


There are more motorist in Greatport than in Fairmont.

We know the number of collisions in Greatport are fewer. Therefore if there were an increased number of
motorist and all other factors remained contant between Greatport and Fairmont, then this would
STREGTHEN the conclusion rather than weaken it.

C. This actually weakens the conclusion as more people reporting the accident = more money the insur-
ance company has to pay.

D. Irrelevant. The comparison is between G and F.

E. Irrelevant. This assumption neither weakens nor strengthens the conclusion.

13.

This is a no brainer. But let us start by negating options.


A. CORRECT.
People in western Africa developed staple crops that they stopped cultivating once rice

and yams were introduced.

This at once destroys the conclusion that by establishing when rice and yams were introduced in Africa
we could determine when the agricultural societies began.

B. Let us negate this

There are plants native to western Africa that, if domesticated, could serve as staple food

crops.

Great. But do we know if these native plants were cultivated by the people in Western Africa? Since we
do not know the answer to this, the conclusion could still be valid.

C. This is good information but neither strengthens or weakens the conclusion as we are specifically con-
cerned with the Western African socities.

D. ok but how does it help with arriving at the conclusion?irrelevant.

E. We already know this. But we also know that these are supplementary food and that no agricultural
society can exist without staple crops.

14.
year x: tax price went up by eight cents. sales fell by 10%
Year before x: tax price constant. Sales fell by 1%

Conclusion: the reason for the sales to fall from 1% to 10% is the 8 cent increase in the tax price.

A.Irrelevant.

year before x : pre-tax price went up. Tax price constant. Sales fell by 1%
year x: pre tax price same as previous yeatt.tax price went up by eight cents. sales fell by 10%
year after x: pre tax price increased slightly but not as much as year before x.

Clearly this does not provide any correlation between increased tax price and decrease in sales.

B. Let us negate this:

The one percent fall in cigarette sales in the year prior to tax increase was not due to a smaller tax in-
crease.

We know that the sales have decreased by 10% this year.

Even if the sales in the previous year HAD NOT decreased due to a tax increase, the conclusion “he rea-
son for the sales to fall from 1% to 10% is the 8 cent increase in the tax price.” still holds. Incorrect.
C. But why did the sales fall to 10% DURING the year of tax increase? Option C does not discuss this at
all and only talks of years before and after tax increase.

D. CORRECT.

Let us negate this:

For the year following the tax increase, the pretax price of a pack of cigarettes was eight or more cents
lower than it had been the previous year.

This would mean that the increased tax price was cancelled out by the decrease in the pretax price. So
the overall price of a pack of cigarettes was lower or the same as before. This would destroy any relation
that exists between increased tax price and the decrease in sales (as the total price for the customer re-
mained the same)

E. Even if the pretax price did not rise and was constant, the overall price would have gone up and this
could be sufficient reason for the decrease in sales.

15.
A. Irrelevant. The comparison is between jobs that require a degree and jobs that don't. Double. triple de-
gree holders and their income is not relevant here :)

B. the comparison is between now and four years later. Not several years ago and the present.

C. CORRECT.

If the negation of this were to be true then once the scarcity no longer exists the high paying jobs wont
exist either and the average salary may not increase.

D. let's say the average salary for non-degree jobs will increase over the next 4 years.

this would actually STRENGTHEN the argument (!), because it would provide another reason that the
average salary would increase.

this is the complete opposite of the effect you're looking for; reversing the assumption should destroy the
argument.

E. Irrelevant.

16.
Conclusion: people who have suffered a serious stroke on the left side of the brain without suffering any
such impairment must have their language centres in the right half.

A. This maybe. But we are only concerned with whether a stroke affects the linguistic capabilities of a
person.

B. Irrelevant. Once again, we are concerned with people suffering from stroke and that affecting their lin-
guistic capabilities.

C. Irrelevant to the conclusion.


D. CORRECT. Let us negate this:

If there are language centres on the left side of the brain, any serious stroke affecting that

side of the brain damages does not damage any of them.


This would destroy the conclusion. It would now be possible for a person to have the language centre in
the left half, suffer a stroke that affected the left half of the brain and still retain linguistic capabilities

E. Since we are discussing people who have suffered a stroke which has damaged the brain, this option
does strengthen the conclusion to an extent.

However, It is NOT A NECESSARY assumption. The negation of the option would still cause the conclu-
sion to hold.

17.
A. CORRECT.

Let us negate A.

The quantities of dissolved salts deposited by rivers in the Earth’s oceans have been unusually large dur-
ing the past hundred years.

The conclusion says that by determining the increased salt levels in the ocean in the past 100 years and
extrapolating it to how many centuries how such deposition would have caused the salt level in the ocean
to rise to the current level, we can determine the age of the planet.

However, if there has been unusually large amounts of salt deposited in the last 100 years, this extrapola-
tion would be inaccurate and the resulting age of the earth maybe far younger.

B. This is not relevant to the conclusion. We are concerned with how much salt these rivers deposit and
not how much they contain. All rivers may have similar salt content, But have they deposited a constant
amount in the ocean?

C. This would weaken the conclusion by indicating that there are other ways in which the salt levels of the
ocean can increase.

D. This is irrelevant. We are discussing the effectiveness of the method at hand. Even if there were supe-
rior methods,our conclusion will remain unchanged.

E. To estimate the age of the oceans one will divide "ocean's current salt levels" A by "the resulting in-
crease in salt levels in the oceans over the past hundred years" a.

If the biological activities are using up salt and have been doing so from the beginning at a constant rate
then we have:

If biological activities use some of the salt, both quantities A and a will decrease correspondingly and the
calculation should lead to the same result.
Whether or not "the salts carried into the Earth’s oceans by rivers are used up by biological activity in the
oceans" will not change the reasoning.

18.
A. Irrelevant to the conclusion.

B. Increasing fertility rates are once again,irrelevant. We want to know if the fertility rates are affected by
the material in the tranquiliser.

C. CORRECT. Let us Negate this:

Any stress that female rhinoceroses may suffer as a result of being immobilized and handled

has negative effect on their fertility.

This would give us an alternate reason for the fertility of the female rhinos decreasing on being immobi-
lised. As you can see, with the negation, the conclusion “Probably, therefore, some substance in the tran-
quilizer inhibits fertility” instantly crumbles.

D. Since we are discussing fertility, we can safely ignore the male rhinos!

E. Since we do not have any information about the fertility rates in these mammals,this option does not
help/harm the conclusion.

19.
The conclusion states that because the modern day analysis indicates low gold content and because the
mining expeditions of the 16 century could not find any gold, the “methods used to determine the gold

content of Frobisher’s samples must have been inaccurate”

A. This weakens the conclusion to an extent by indicating that there was more gold on the island in 16th
century than presently.

B. This weakens the conclusion to an extent by indicating that since the same parts of the island were not
mined, they perhaps missed the gold rich soil.

C. Different does not necessarily mean inaccurate or less effective.

D. Irrelevant. We are discussing Kodlunarn Island only.

E. let us negate this


Gold was added to the soil samples collected by Frobisher before the samples were examined.

This would destroy the conclusion that the technique used by Frobisher was inaccurate and infact indi-
cate that while his technique was correct,the soil had been tampered with to indicate high gold content.

20.
A. This maybe. But the goal was to decrease the hazardous waste by half. This option does not tell us
anything about hazardous waste.

B. is irrelevant, as we have no information about how much waste is produced PER JET - not now, not
then.

C. once again , this is irrelevant. The goal was to “reduce by half the total yearly amount of hazardous
waste generated by its passenger-jet division”.

The other divisions are irrelevant to our argument.

D. We are discussing the waste output per pound per worker. The amount of time they spend working is
irrelevant

E. CORRECT.
the production company's GOAL was to lower the TOTAL AMOUNT of hazardous waste produced.

the EVIDENCE is stated in terms of the amount PER WORKER, not the total amount.

Let us negate this option


“ The number of production workers assigned to the passenger-jet division was significantly less in 1994
than it was last year”

This tells us why the amount of waste per worker may have gone down despite the amount of hazardous
waste being produced having remained the same or increased. This would cause the conclusion to crum-
ble.

21.
This one is pretty simple. Let us try our favourite method of negation here:

A. Irrelevant. This does not answer whether cannabinoids help stimulate the appetite or not.

B. The passage does not require us to assume that cannabinoids are the ONLY substance that stimulate
the appetite. Even if there were other substance that do so, the passage goes on to tell us that blocking
cannabinoids caused the appetite to decrease. This option does to give us any reason why this may have
happened.

C. Irrelevant. This was true both before and after the mice were injected with a chemical that blocked
cannabinoids.

D. Irrelevant. Does it help stimulate appetite? Thats all we care to know.

E. CORRECT.
Let us negate this.
“The chemical that blocks cannabinoids from stimulating their brain receptors independently inhibits the
appetite”

This clearly suggests that the loss of appetite was caused due to factors other than absence of canna-
binods. This destroys the conclusion.
22.
Let us assume that before 1992 there were 100,000 catchable lobsters in the sea.
Of 100,000 catchable lobsters, let us assume all 100,000 were legally harvested.

In 1996,

we know that the number of legally harvested lobsters were 91,000.

Therefore the passage states that that 9000 of the remaining lobesters (assuming there are still 100,000
catchable lobsters) were caught by illegal activities.

A. CORRECT.

let us negate this


The illegal lobster harvesting was so extensive that the population of catchable lobsters in

Belukia’s territorial waters had sharply declined by 1996.

This tells us that the catchable population is no longer 100,000.

Let us assume its come down to 95,000. So going by this assumption, the number of lobsters illegally
harvested comes to 95,000 – 91,000 = 4000. This destroys the conclusion.

B. Maybe. But do we know if it has increased to 9000 tons?

C. Irrelevant. We already know this to be true from the passage.

D. Since the annual legal catch was 9000 tons less than the pre 1992, we already know this statement to
be true.

E. This one may seem tempting. But we know that there “no reduction in the level of legal lobster fishing
activity”. Thus this option does not affect our conclusion.

23.
conclusion: Since the hotel taxes were lowered and yet the govt collected the same amount of money
from thee hotels,it follows that more number of tourists stayed in Midville that year.

A. Irrelevant. While the tourists maybe aware of lowered hotel rates, the option does not tell us that more
tourists came in because of it.

B. The prices in other cities is irrelevant.

C. CORRECT. Let us negate this:


“The average length of a tourist’s stay in Midville hotels was longer last year than it had been the year
before.”
This tells us why the despite the number of tourists having remained the same or lesser than previous
year,the hotels could have made more money thereby destroying the conclusion.
D. Irrelevant. The argument states that lowered taxes led to more number of tourists staying at Midville.
We are not discussing Midville's marketing stratergy here.

E. Tempting but please note that we are discussing lowered tax on hotel accommodation here. The price
on the meals is irrelevant.

24.
This is pretty straightforward.

Premise: one candidate produced a half-hour-long advertisement. During the half hour the advertisement
was aired, a substantial portion of the viewing public tuned into the advertisement.

Conclusion:many more people are interested in watching lengthy televised political messages than was
previously thought.

A. we are not interested in the effect the advertisement had on the people. Our only concern is do people
want to watch the long advertisement or not. If we negate this statement “ The candidate’s ratings did not
improve significantly as a result of the half-hour-long political advertisement” the conclusion can still be
valid. Maybe people did tune in to watch the campaign but did not like what they saw.

B. Irrelevant. We are concerned with the length of the political message.

C. Thats good. But do they want to do so in the form of lengthy televised political message?Thats the
crux of our argument that this option does not address.

D. Irrelevant. We already know a good amount of the population tuned in to watch the ad.

E. CORRECT.
Let us negate this:

Most of the viewers who tuned in to the candidate’s half-hour-long advertisement last year changed
channels after the first few minutes.

This would immediately destroy the conclusion that more people are interested in watching lengthy politi-
cal messages.

25.

This one is quite easy. Once again, let us refer back to our old friend-negation

A. This actually weakens the conclusion stating that the nerve damage caused by other chemicals would
be different from that caused by EB, indicating that we would be able to distinguish the reason for the
nerve damage.

B. Irrelevant. Does EB cause nerve damage or do the new chemicals being used also cause it?This op-
tion does not help us confirm either of the questions.
C. CORRECT. Let us negate this.
If ethylene dibromide causes nerve damage, it takes two years or longer for that

damage to become detectable.

If this were true then we know that EB was NOT wrongly blamed and also that new chemicals do not
cause nerve damage. This would collapse the conclusion.

D. we are talking about people who were “newly diagnosed”

E. strengthens the argument to an extent. But we will still have to assume that these plants do not have
any external factors that could cause nerve damage.

26.
This problem may seem tough only due the way its been worded.
The summary, simple put is this: There are some people who are allergic to sulphites. Several wine mak-
ers do not add sulphites as preservatives to their wines.

Conclusion: therefore people suffering from allergic reactions can drink wine from such makers to avoid
allergic reactions.

A. We are only concerned with sulphites and the allergy induced by them. We are not discussing any oth-
er “potentially allergic” substances.

B. We clearly know that people are allergic to sulphites that are added to preserve wine. That makes this
option irrelevant.

C. This information is irrelevant.

D. Again,we are discussing allergic reactions to sulphites in particular. Not relevant to the conclusion.

E. CORRECT. Let us negate this.

Sulfites are naturally present in the wines produced by these wine makers in amounts large enough to
produce an allergic reaction in someone who drinks these wines.
This clearly destroys the conclusion that such wines can be consumed by people with allergic reaction to
sulphites.

Therefore option E is a critical assumption we need to make to arrive at this conclusion.

27.
A. Irrelevant. The comparison is between households with a built-clock in their microwave oven and
household that have just the oven. The comparison is not between households with ovens and those
without.

B. This information is irrelevant to the discussion. We are discussing the electricity consumption.
C. Additional information to the stimulus - not helpful in addressing the issue at hand. Even if the nega-
tion of this option were true, the conclusion will hold.

D. CORRECT. - stating that if an oven don't have a clock, the household will not install one is essential to
the conclusion that houses without built in clocks in the microwave consume less electricity. The negation
of this option would destroy the conclusion “Therefore, households whose microwave oven has no built-in
clock use 45 kilowatt-hours per year less, on average, than do comparable households whose microwave
oven has one”

Negation:

Households whose microwave oven does not have a built-in clock are more likely to have a separate elec-
tric

clock plugged in than households whose microwave oven has one.

E. - Additional information to the stimulus - Not quite relevant. True otr not, this option does not affect
the conclusion.

28.

The conclusion is : “Therefore, by selling the helium, the government can not only pay off that debt but
reduce its overall debt as well”

Also its important to note that currently the govt can make 25% more money by selling heilum than the
current debt incurred due to it

Let us see which options support this conclusion.

A. This maybe. But does it mean that by selling the helium the govt will be able to pay off debts?We are
not told.

B. Whether it is a significant portion of the total debt is irrelevant. We know that by selling of helium, the
government can reduce its helium debts as well as a portion (whatever that maybe) of its total debts

C. This is completely out of scope.

D. CORRECT.

Let us negate:
Attempts to sell the government’s helium will depress the market price of helium by more than 25 percent.
This would mean that the prices of helium would fall by the 25% margin and not allow the government to
recover the debts it has incurred in purchasing and storing helium.

E. For the conclusion to fall apart, the loss incurred should be 25% more than the money spent in acquir-
ing and storing helium. This seems highly unlikely and moreover is an assumption thats outside the pas-
sage.
29

Premise: a technology for preventing leaks of oil pipelines is being installed


Conclusion: as long as the above technology is effective,there will be no leaks and hence no danger to
the fish.

Assumption: Are the leaks the only thing that may endanger the fish population when oil pipeline is con-
structed?

A. The conclusion talks of the dangers of constructing oilpipelines only. This is out of scope.

B. CORRECT.
Negation: Other than the possibility of a leak, there is a realistic pollution threat posed to the lake by the
pipeline’s construction.

This would mean that even if the anti leak technology was successful it would still destroy the fish popula-
tion thereby causing the conclusion to fall apart.

C. This is already stated in the conclusion “provided the technology is effective” so we know that the con-
clusion is subject to the condition that the anti leak technology is successful. This neither strengthens nor
weakens the conclusion.

D. Irrelevant. We are not concerned about other damages.

E. The passage discusses fish population in general and this is not specific to a certain species.

30.
Summary:

Coponia plans to increase the tax by 9 cents. This would inturn cause a 10 percentage point increase in
the price of cigarettes.

A 10 percentage point increase in price has always caused the sales to Dip by 4%.

conclusion: therefore,rising the tax by 9 cents on a 90cent packet of cigarettes will also cause the sales to
decrease by 4%.

Note: it is essential that the overall price of the cigarettes increase to cause a decrese in sales. What if
the cigarette companies decide to cut down their profit margin and absorb the extra 9 cent costs on the
tax to prevent reduction in sales? In this case the overall price of the cigarettes will not rise and the sales
will not fall.

Therefore option A is correct.

31.
The main conclusion here is by offering a rigorous fitness program of jogging and weight lifting to all em-
ployees, and requiring employees who are out of shape to participate, the company will save doctor bills
and longer hospital stays that less fir employees usually incur.
Therefore its more profitable for the company to invest in a fitness program.

A. This is irrelevant to the conclusion. We know already that less fit people incur higher medical bills.

B. This weakens the conclusion by stating that somehow the people asked not to participate in the pro-
gram incur heftier medical bills. This is a complete contradiction of what's stated in the passage.

C.CORRECT.
Let us negate it:
The strenuous activities required of out-of-shape employees by the program would by themselves gen-
erate medical expenses greater than any reduction achieved by the program.

This means that the fitness activity for our of shape employees will cost the company a whole lot more
interms of medically bills and more importantly,would be “greater than any reduction achieved by the
program”
This completely destorys the conclusion

D. We already know this to be true.

E. While this a good thing, it says the lesser fit employess will be at work more than the more fit employ-
ees(not required to participate in the program) This does not directly explain why the health insurance
cost for the company will go down.

32.
If this is solved based on the negation technique A is a clear choice
Negation of A :The tetracycline deposits formed after the bodies were buried.

This destroys the conclusion that “tetracycline in their food probably explains the low incidence of typhus
among ancient Nubians”

33.

Since the number of shops and workshops in Brindon County have reduced, the passage states that the
output of Brindon Bolt Barn will also reduce.

Let us see the negation of which option would destroy this conclusion/

A. This is irrelevant. The passage tells us that the workshops were shut down in the recent months and
the sales and revenue for next year will decrease.

B. Although this tells us to some extent that the amount of work for the workshops in Brindon County is
not very high,therefore sales may not be much, we do not know if the Brindon Bolts operates only in Brin-
don County or outside of it.

C. We are concerned with the sales and revenue of Brindon Bolts Barn only.

D. This is out of scope. We need information about its sales and revenue.
E. CORRECT. Let us negate this:
The Brindon Bolt Barn is a company that gets the great majority of its business from customers outside
Brindon County.

This means that although shops have shut in the Brindon county and its sales in the county maybe low,
the overall sales of Brindon Bolt Barn will continue to be high as compared to last year.

34.

Conclusion: Since average homework time is 30 minutes, there is no need to impose limits on the amount
as it does not eat into much of children's free time.

A. Even if this weren't true, the theorist's argument is that the children will have sufficient time to complete
their homework as well as engage in free time activities.(irrespective of what those activities are)

B. This argument isn't about the effectiveness of homework. Rather its about whether a child can do
homework as well as have time for other activities.

C. This would actually weaken the theorist's argument to an extent indicating that we need someone else
other than teachers to decide the amount of homework (Exactly what he is arguing against)

D. CORRECT. Let us negate this:

In most schools, if not all, the homework assignments given are of a length that diverges widely from the
average.
This would mean that children may have to spend more than 30 mins which might cut into their free time
and thereby proving the point made by the editorials. This would destroy the conclusion.

E. The theorist does not state that free time activities are useless. This is out of scope.

35.

The crux of this argument is as below:

Public advocacy groups can directly pass on information to the public. This means that information ser-
vices(the internet) present a more balanced picture of the complexities of political issues than any tradi-
tional news source presents

A. the argument does not discuss the reach of information services or newspapers. It simply says that
through information services a more balanced picture can be given to the public.

B. This actually weakens the conclusion by stating that an unbiased view can also be obtained by sorting
through several papers.

C. Correct. Let us negate this:


Information on political issues disseminated through information services comes almost entirely from ad-
vocacy groups that share a single bias.

This would mean that despite using information services the information will still be as (or more) biased
that the ones got in the papers,defeating the purpose. This would effectively destroy the conclusion.

D. We already know this to be true.

E. Once again this weakens the conclusion by indicating that there is no need for an unbiased source.

36.

A. CORRECT.

Here's a diagram for this argument:

(# of rds-equipped stations increased 250-->600)

BUT

(apprx same # of rds radios in Vland)

THEREFORE

(apprx same # of people receiving rds signals in Vland)

if you make this diagram, it should be clear that there's a logical leap between the latter two statements:
the author is assuming a direct correlation between the # of rds-equipped radios and the # of people
who actually receive rds signals with those radios.

in order to make this connection, you need to assume that nobody, or almost nobody**, with an rds-
equipped radio is now (in '96, that is) able to receive a signal but wasn't able to receive a signal back in
'94. that's pretty much what A. says.

B. Irrelevant. As it does not directly discuss whether these people already had a RDS radio before the
station was built , it does not fill the gap.

C. C is out of scope. It says that equipping a radio with the RDS feature does not decrease the range of
the radio. But who cares? Even if it did, this would have been the case for 1994 as well as 1996. We
need something that connects the data about radios and 94-96 with the claim that THE SAME NUMBER
OF PEOPLE received the programming, even though no one bought new radios.

D. D is not an assumption - it is a fact. We are told that you cannot get this type of signal without the
right equipment. An assumption must be something that is unstated.
E. Let’s negate this:

The RDS radio stations in Verdland in 1996 all offered the same type of programming.
This does not affect our conclusion in any way and hence is irrelevant.

37.

a) Reverse this assumption:

The Sepphoris mosaics are composed exclusively of types of stones found naturally in teh Sepphoris ar-
ea
This is consistent with the conclusion. Note that the passage says the mosaics were “created” by travel-
ling artisans which means that they could have used the locally available stones at Sepphoris to create
the mosaics.

B.

reverse this assumption: let's say all the species are indigenous to some common region.

this doesn't destroy the argument; it's perfectly consistent with the idea of traveling artisans (who presum-
ably would have come from that common region).

wrong answer.

C. reverse this
motifs that appear in the Sepphoris mosaics that also appear in the mosaics of some

other Roman city

This actually strengthens the conclusion rather than destroying it.

D.

reverse this assumption: let's say that there are some animal figures that are not readily identifiable.

this has no effect whatsoever on the argument, which is concerned only with some of the animal figures
(i.e., the ones that weren't native to the local area).

wrong answer.

E. CORRECT

reverse this assumption: let's say there was a common repertory of mosaic designs.
in this case, that repertory - since it was a common repertory - would have included animal figures from all
over the place. (at the very least, it would be quite unreasonable to expect a common repertory to have
been restricted to animal figures from the sepphoris area in particular.)

this destroys the argument, because, were there such a repertory, then artists local to sepphoris would
have followed it as well, creating the exotic designs despite their status as natives in the area.

38.

the first premise states, as fact, the following correlation:

higher vitamin/minerals in bloodstream <--> better lung function

The loophole:

if smokers get higher levels of the vitamins/minerals into their bloodstreams, then based on the premise
they will also have better lung function. The argument is therefore airtight, provided the smokers can get
the vitamins and minerals into their bloodstreams to begin with.(absorption)

That's the only missing link here - all that's mentioned in the argument is intake, which is an entirely sep-
arate matter from absorption into the bloodstream. We need to look for an option that can close this gap.

A)Irrelevant. it speaks of vitamins and minerals in general, not the special vitamins and minerals men-
tioned in the passage.

B)Strengthens the premise which we already know to be true

C)Irrelevant as we are only discussing how lung capacity of a smoker can be improved. People suffering
from vitamin deficiency (smokers or non smokers) who have other health problems are not relevant here.

D)irrelevant, as cessation of smoking doesn't affect the issue (the effect of the vitamins/minerals) at all.

E)CORRECT because it takes care of the connection between intake and absorption, by establishing that
the smokers' intake of vitamins and minerals will actually make it into the bloodstream.

Let’s negate this option:

Smoking introduces into the body chemicals that prevent the helpful vitamins and minerals from entering
the bloodstream.
This would destroy the conclusion by stating that despite smokers increasing their vitamin intake it will
not help improve their lung function.

39.

A. Let’s negate this:

"The “Thank you” messages would not have the same impact on regular patrons of a restaurant as they
would on occasional patrons of the same restaurant".

This means that the message must have a positive effect on group and a negative effect on the other.

Even if the messages do not have the same impact on regular patrons as they do on occasional patrons,
the average income from tips would still increase. The group on which the messages have had a positive
effect will continue to tip more.

B. CORRECT.

Let’s negate this:

"Regularly seeing “Thank you” written on their bills would lead restaurant patrons to revert to their earlier
tipping habits."

This means that the reason the patrons tipped more was because the messages were a one off thing.
Seeing the message with the bill regularly will not have the same impact anymore.

Therefore the assumption that seeing the message regularly will not cause the patrons to revert to their
previous tipping habit is essential to the conclusion.

C. We already know that the patrons are tipping more. The reason that they are doing so is not relevant
to the conclusion and is definitely not the assumption on which the argument relies.

D. The argument says that the "Tips on these bills were an average of three percentage points higher
than tips on bills without the message."

Since the comparison is within the same hotel, between bills with messages and bills without, even if rate
at which people tip food servers in Pennsylvania varied with how expensive a hotel was, it still does not
destroy the conclusion.
E. We know that there was an an average three point increase in the tips. Even if this option weren't true
i.e "Not all patrons of the Pennsylvania restaurants in the study who were given a bill with “Thank you”
written on it left a larger tip than they otherwise would have."

the conclusion still holds.

40.

A. Many scientists make this assumption, of course – but its not an assumption on which the explanation
depends. The explanation concerns the scientists' motivation, not their style of doing research.

B. This statement gives another reason that scientists may reject a populariser, but because it is not the
reason implied in the passage, it is not assumed.

C. Even if this is true, it does not address the core issue of the argument, what scientists believe about
the motivation of popularisers.

D. CORRECT. This statement properly identifies an assumption on which the explanation for scientists'
rejection of popularisers depends.

E. The passage is not concerned with whether non-scientists can understand new research, but rather
with the beliefs and motivations of scientists who reject popularisers as colleagues.

41.
A. The argument concerns innovation in bicycle technology. It is not about the entire market for all bicy-
cles, so this claim about traditional bicycles is not assumed.

B. The passage does not discuss where the best innovation are likely to be created, so no assumption
about small workshops versus large manufacturers is made

C. CORRECT. This statement identifies information that appropriately fills the gap in the reasoning as
stated in the passage.

D. This claim provides an explanation of manufacturers' technological conservatism that is quite different
from the explanation indicated in the passage.

E. The passage does not indicate what the racing authorities do or do not know about bicycle innovation
– even though it suggests that they may be reluctant to approve every possible innovation for racing
purposes.
42.
Sharon makes a general statement claiming that if a person knows 50 workers,it is likely that at least one
of them is unemployed. This generalization is not likely to be true if unemployment were concentrated in
certain geographic isolated areas.

A. Sharon's argument is about normal level of unemployment; how rarely or frequently that level is ex-
ceeded is outside the scope of her argument.

B. CORRECT. This statement properly identifies an assumption that underlies Sharon's argument If un-
employment were concentrated in geographically isolated segments of populations, then randomly se-
lected individuals would not have an equivalent chance of knowing an unemployed person. Instead,
based on geographic factors, some individuals would be much more likely to know unemployed workers
than would other individuals. In this case, Sharon's conclusion crumbles.

C. Although Sharon's argument is compatible with saying than even more than 90% of the population is
unemployed, nothing suggests that she assumes this is true.

D. Sharon's argument is not based on the figure Roland cites and does not assume its accuracy or inac-
curacy; her argument merely points out that his figure is not consistent with a normal rate of unemploy-
ment.

E. Irrelevant. We do not know if such a fear has any consequence. Maybe the person will work harder to
keep his job or maybe he will bungle it up. The negation of this option would still cause Sharon's conclu-
sion to hold.

43.
Premise: Because no employee wants to be associated with bad news in the eyes of a superior, infor-
mation about serious problems at lower levels is progressively softened and distorted as it goes up each
step in the

management hierarchy.
Conclusion: The chief executive is, therefore, less well informed about problems at lower levels than are
his or her subordinates at those levels.

Assumption: The chief executive is less well informed about problems because his subordinates soften
and distort the information at each level( based on the premise) indicating that problems have no other
way of reaching the chief executive other than through his subordinates.

A. is tempting to choose because it seems like a pretty good assumption to make in the real world; An-
swer A is wrong, though, because how the problems “should” be solved doesn’t tell me anything about
how well-informed the chief executive is about those problems.

B. is tempting to choose because it sounds like a good way to resolve the problem described in the ar-
gument. Answer B is wrong, though, because we weren’t asked to resolve the problem; we were asked
to articulate a belief (an assumption) of the author who is pointing out the problem.
C. is tempting to choose because it sounds like a pretty good assumption to make in the real world. An-
swer C is wrong, though, because the ability to solve a problem still doesn’t tell me anything about how
well-informed the chief executive is about those problems.

D. CORRECT. because this is exactly the assumption that the author makes to draw his conclusion. If
CEOs really can’t get info from anyone other than their subordinates, and if those subordinates don’t
want to tell them any bad news, then those CEOs are not going to be well-informed about problems.

E. wrong, because it weakens the argument: if some subordinates are speaking up, then the bosses
aren’t less well-informed. We were asked to find an assumption, and an assumption is something the au-
thor must believe to be true in order to draw that conclusion. If the answer choice actually weakens the
conclusion, then that answer can’t be a valid assumption

44.
Conclusion: Personalities unsuited to the requirements of the job will be eliminated from consideration
during the interview.

Assumption: such personalities can be identified during the interview process.

We need to look for an option that addresses this assumption.

A. This is simply restating the premise. We already know that “interview is an essential part of a success-
ful hiring program”.

B. There is nothing in the passage that makes the comparison between job development and interview.

Option C is CORRECT.

“Interviewers can accurately identify applicants whose personalities are unsuited to the requirements of
the job”
Clearly addresses the assumption made by the argument.

Let’s try negating this option:

Interviewers cannot accurately identify applicants whose personalities are unsuited to the requirements
of the job”
Now the entire conclusion made above crumbles, if the interviewers cannot identify suited/unsuited per-
sonalities for the job then the purpose of an interview is lost. This proves option C is CORRECT.
D. It can be the only factor or it can be one of the many factors assessed in an interview. All we know is
that it is a factor. The conclusion will remain strong irrespective of whether the negation of this statement
is true.

E. We are not concerned with what was important in the past.

45.
Fact:Bank depositors in the United States are all financially protected against bank failure because the
government insures all individuals' bank deposits
Premise: Since depositors do not lose any money of the banks fail ,they do not care about the soundness
of the investment the bank makes.
Conclusion: If the government did not insure the depositors money, the depositors would take more care
in understanding the bank polices, thereby making the banks more secure.

This conclusion is clearly based on the assumption that “the depositors would take more care in under-
standing the bank polices and hence will be able to determine which is a safer bank ”.

Let’s look for an option that strengthens this assumption.

A. How bank failures are caused is irrelevant.

B. Irrelevant. The passage does not assume the number of accounts per bank per depositor anywhere.
C. Once again, the passage does not assume this. There is no connection made to the amount deposited
versus the care taken by the depositor to pick a bank.

D. Irrelevant. Even if the opposite were true I.e “The difference in the interest rates paid to depositors by
different banks is a significant factor in bank failures” it would still make no difference to the economist's
argument which indicates that depositors become complacent on being assured of their money back de-
spite bank failures.

E .CORRECT. This is the basis of the conclusion made by the economist. If potential depositors are una-
ble to determine which banks are secure against failure, then the entire conclusion that removal of gov-
ernment insurance will make banks safer would crumble.

46.
A. This is irrelevant. We do not care about commercial fish . Our concern is the porpoise alone.

B. CORRECT

negating it , we have

“ When disoriented, harbor porpoises are significantly more likely to be killed by other boats.”
This would destroy the conclusion that “The alarms, therefore, are saving the

lives of harbor porpoises in those waters.”

C. Although this might strengthen the use of alarms, its not an assumption. This is simply a fact stated iin
support of the plan i.e the premise.

D. Irrelevant. Even if the harbor porpoises were not the largest in number at the time the alarm was in-
stalled, it does not affect the conclusion.

E. We talking about the safety of the porpoises and not the costs involved. This is irrelevant.

47.
An assumption that supports a conclusion, though unstated, is NECESSARILY TRUE. In other words, the
conclusion must directly depend on the information from the assumption. In this argument, the conclu-
sion states that drivers who equip their vehicles with radar detectors are more likely to exceed the speed
limit regularly than are driver who do not. The only factual basis (i.e. premise) for this conclusion is that
33% of vehicles ticketed for exceeding the speed limit were equipped with radar detectors, while only
3% of drivers on Maryland highways equip their vehicles with radar detectors.

The only information provided is about the percentage of vehicles equipped with radar and the percent-
age of ticketed vehicles equipped with radar; the conclusion, however, makes a sweeping generalization
about the regular driving behavior of individuals with radar-equipped vehicles. When the GMAT creates
such a "jump" or "gap" in the logic of an argument, the assumption will often "fill in" the missing infor-
mation.

In order for the conclusion to be valid, it must be true that these ticketed drivers exceed the speed limit
on a regular basis. This is satisfied by option B.

48.
The argument very clearly assumes that the newly thin people will continue to consume the same num-
ber of calories that the people whose weight is normally at that level do. Thereby they are unable to burn
as many calories as the people whose weight is normally at that level and regain their original weight.

A. CORRECT. Negating this we find “Large number of very overweight people who have dieted down to a
new weight tend to continue to consume substantially fewer calories than do people whose normal
weight is at that level.” this would make our assumptions untrue and would allow the very over weight
people to maintain their new weight.

B. Negation of B
the metabolism of people who are usually not overweight are not able to vary than the metabolism of
people who have been very overweight.

Since the argument has already established that the metabolic rates of the usually overweight don't much
vary with dieting, the real force of negating B is the metabolic rates of those who are not usually over-
weight don't much vary with dieting But this doesn't weaken the conclusion that the usually overweight
will regain weight. It doesn't say anything whatsoever about the usually overweight.

C. Negation “The amount of calories that a person usually burns in a day is not determined more by the
amount that is consumed that day than by the current weight of the individual.”

This simply tells us that the amount of calories burned depends on the current weight of the individual I.e
the metabolism or equally on the amount eaten as well as the calorie intake.
However,the premise already tells us that the metabolism of overweight individual that has lost weight
through dieting is lower than that of an individual who is normally at that weight.

D. Irrelevant. Whether such a chemical exists or not, it will in no way affect the conclusion which dis-
cusses people who have lost weight primarily through dieting.

E. Irrelevant. The conclusion talks about people who are overweight and have lost weight,not people with
normal weight.

49.

the entire argument can be summed up as follows:

top managers use intuition more than do midlevel or lower-level managers

-->

intuition is MORE EFFECTIVE

there's a huge, glaring hole there: the argument has connected "top managers use intuition more" with
"it's a more effective way of decision making". these 2 specifics are entirely different from one another,
and NEED to be bridged for the argument to hold up.

in fact, you should get into the habit of regarding ANY two specifics that are even remotely different
(such as "drivers who speed" and "drivers who receive speeding tickets") as completely different things.
under that criterion, it's a no-brainer to choose the assumption that bridges the gap.

A. Irrelevant. The passage already suggests that Methodical, step-by-step reasoning is employed by
managers for decision making.
B. Irrelevant. “top managers used intuition significantly more than did most middle- or lower-level man-
agers “ indicates that both top level managers and middle level managers have the ability to use either
intuitive reasoning or methodical, step-by-step reasoning in making decisions.

C. Irrelevant. Even if this were true, it wouldn't explain why “intuition is actually more effective than care-
ful, methodical reasoning”

D. Irrelevant. The comparison is between the success of the methods used by top managers versus mid-
dle level and low level managers.

E. CORRECT. This explain why intuition is considered a more successful way of decision making than
step-by-step reasoning by indicating that the managers who employ the former (Top managers) are
more successful.

50.

The argument concludes that since the evaluated cookware is superior to all other cookware advertised in
Kitchen, lending the magazine name to the line of cookware will DECREASE the revenue of the magazine
as the reputation of the magazine will not suffer.

The argument assumes that reputation alone is responsible for generating revenue.

Let’s use the negation technique to find which option destroys the conclusion.

A. Negation:

Other line of cookware is superior to that which will carry the Kitchen name.

This does not affect the conclusion. We are clearly told that among all the cookware advertised in Kitch-
en, the one Kitchen is lending it's name to is of the highest quality. Therefore it is unlikely that the read-
ers of Kitchen will be put off by a product that is not advertised in Kitchen if it's of a superior quality.

B. Negation:

Kitchen will license the use of its name for any products other than the line of cookware.

Unless we are explicitly told that the quality of these products is below the expected standard, it does not
affect the conclusion.

C. CORRECT

Negation:

Makers of cookware will find Kitchen a less attractive advertising vehicle because the magazine's name is
associated with a competing product.
This gives us a reason why the revenue of Kitchen might decrease despite the line of cookware it is lend-
ing it's name to being of superior quality.

If the competing cookware lines refuse to advertise in the Kitchen, it will lose it's advertisement revenues
from those companies.

D. Negation:

Consumers who are regular readers of Kitchen magazine will be attracted to the cookware by the Kitchen
name.

This would further strengthen the conclusion rather than weaken i.

E. Kitchen is not one of the most prestigious cooking-related magazines.

This neither strengthens nor weakens the conclusion. We are simply concerned with how Kitchen's plan
of lending it's name will affect revenues.

51.

A. This is irrelevant. The question is whether writing VOID on the check will deter the production of coun-
terfeit checks .

B. Once again, this is out of scope. Irrespective of the denomination for which the fake checks are made,
will issuing checks with dots too small to be picked up by the electronic scanner used by counterfeit
checks,help?

C. CORRECT.

This tells us that a layman will not be able to distinguish between a counterfeit check and real one as the
dots used on the real check will be too tiny for the naked eye to catch.

D. This strengthens the plan slightly by telling us that the cost of such new checks will be no more than
the old ones.

E. This also strengthens the plan by telling us that the electronic scanners in the near future will not be
able to duplicate such tiny dots.
Assumptions Part 2

1.

The Police Commissioner's proposal hopes to decrease the number of crimes in city Y by shift-
ing police officers from low-crime to high-crime districts. His proposal is based on data that
demonstrate that crime decreases when additional police officers are moved into a district.
However, the data do not mention anything about the effect on the districts from which the po-
lice officers were removed. The commissioner's plan is based on the assumption that the
movement of police officers will not have any adverse effects on the low-crime districts.

(A) While it is encouraging that a similar plan worked successfully in City X, this fact is certainly
not essential for the success of the plan in City Y. The cities may be so different as to make the
comparison meaningless.

(B) The police commissioner's proposal is focused solely on decreasing the number of crimes in
city Y. The severity of the crimes has no bearing on whether the commissioner's proposal will
succeed or not.

(C) The actual numerical distinction between high and low-crime areas of the city is immaterial
to the commissioner's proposal. For instance, if the number of crimes committed in all high
crime districts was only double (instead of more than triple) the number of crimes committed in
low crime districts, the proposal could still be valid.

(D) It would be practically beneficial to the commissioner's plan if there were more low crime
than high crime districts in city Y. This would enable the movement of police officers to every
high crime district. However, this is not necessary to achieve the commissioner's goal of de-
creasing the total number of crimes in city Y. Even if there were more high-crime districts than
low-crime districts in city Y, police officers could still be shifted to some (though not all) high-
crime districts, and thereby possibly reduce the total number of crimes in city Y.
(E) CORRECT. The police commissioner's proposal would not make sense if districts of the city
from which police officers are removed experience significant crime increases shortly after the
removal of those officers. This would at least partially, if not fully, negate the reduction in the
number of crimes in the high-crime districts. This choice establishes that, in fact, the low-crime
districts do NOT suffer from significant crime increases after the removal of some officers--an
essential assumption upon which the commissioner's proposal depends.

2.

The argument concludes that rising sea levels caused by global warming will destroy major
coastal population centers and displace millions of people. Any assumption in support of this
conclusion would have to corroborate that these events will definitively take place.

(A) CORRECT. If new technological developments in the next century allow people to divert
rising seas from the world’s cities (i.e., population centers), cities will not be destroyed and mil-
lions of people will not be displaced. Thus, a necessary assumption is that these technologies
will not be developed.

(B) A simple awareness of the steps to reduce emissions in no way undermines the argument’s
conclusion, as this answer choice does not describe any action being taken by individuals. Addi-
tionally, greenhouse gases are never mentioned as the primary by-product of human activity
that causes global warming, and are therefore not sufficient to address the argument.

(C) The argument never suggests that all coastal population centers are similarly affected; this
choice is too extreme and overreaching for the argument’s conclusion.

(D) This might be true, but it is not an assumption on which the conclusion rests. Instead, this
answer choice is simply an inference that might be drawn from the premises.

(E) The idea that human activity is the sole cause of global warming is neither suggested nor
assumed by the argument. In addition, the wording "sole cause" is too extreme.
3.

As an advertisement, this passage attempts to entice the reader into purchasing a new HitItFar
driver by touting its benefits, both implied or explicit. The key to answering this question is to
be able to analyze each claim to determine whether it is implied, explicitly stated, or neither.

(A) By asking the reader rhetorically "isn't it time you added power ... and distance ... [by
switching to the HitItFar driver]?", the advertisement implies that the use of the club will add
"power ... and distance" and, hence, will improve one's play.

(B) CORRECT. The advertisement states that the 12 major championships winner have recently
switched to the new driver. There is nothing in the passage to imply that any of them were us-
ing the driver at the time of their victories; hence, this claim is neither implied, nor made explic-
itly, in the passage.

(C) It is a reasonable assumption that professional golfers, particularly those skilled enough to
win a major championship, are experts and know what constitutes a great club. The advertise-
ment makes this implication and reinforces it by citing the recent decision of these golfers to
switch to the HitItFar driver; this provides an implicit expert endorsement for the HitItFar driver.

(D) The point of the advertisement is to prompt the reader into purchasing a new driver to re-
place his or her old driver in order to "add power and distance"; this implies that the new driver
is superior to the reader's existing driver.

(E) This claim is explicitly stated in the first sentence of the passage.

4.

The researchers claim that Delta-32 prevents its carriers from contracting the Plague. They
support this claim by noting that a strikingly large percentage of descendants of Plague survi-
vors carry the mutation. We are asked to find an assumption underlying the claim.
(A) The argument is specific to the relationship between Delta-32 and resistance to the Plague.
Other diseases are irrelevant.

(B) Again, the argument is specific to the relationship between Delta-32 and resistance to the
Plague. Other diseases are irrelevant.

(C) Delta-32 may have existed in its current form before the sixteenth century and the merit of
the argument would not change.

(D) The argument does not claim that Delta-32 prevents all bacteria-caused disease.

(E) CORRECT. The researchers claim that Delta-32 prevented its carriers from contracting the
Plague on the basis of its presence in descendants of Plague survivors. But it is theoretically
possible that these descendants carry the mutation Delta-32 because the Plague mutated the
genes of their ancestors. In order to claim that the mutation prevented the Plague, we must
assume that the Plague did not cause the mutation Delta-32.

5.

The author concludes that one will only be able to determine the age of a Brazilian ash by
counting its rings if the temperature in the tree's environment never exceeds 95 degrees Fahr-
enheit. The author bases this conclusion on the fact that the tree loses rings when the tempera-
ture exceeds that level. However, if the number of rings lost by a Brazilian ash at high tempera-
tures can be predicted, it may be possible to determine the age of a tree even if the tempera-
ture exceeds 95 degrees.

(A) The argument says nothing about precipitation. This answer choice is out of scope since it
would require a number of other assumptions to make it relevant to the argument's conclusion.

(B) Whether other trees share this feature is irrelevant; the argument focuses only on the Bra-
zilian ash.
(C) The number of days of excessive heat needed to cause the tree to lose rings is irrelevant.

(D) The thickness of the rings is irrelevant.

(E) CORRECT. The conclusion is that the rings will be a reliable measure only if the temperature
never exceeds 95 degrees. This is true only if there is no way to predict how many rings would
be lost when the temperature does exceed 95 degrees. (If it were possible to predict this, one
might be able to assess the age of a tree using its rings even if the temperature had exceeded
95 degrees.)

6.

The conclusion of this argument is that the national identification system (“using licenses for
purposes not directly related to operating a motor vehicle”) is un-American. The basis for this
claim is that such a system would allow the government to restrict the liberty of its people. The
necessary assumption is one that connects restrictions on liberties to the concept of “un-
American” policies.

(A) The author never mentions future presidential elections, or the role of the president in such
a national identification system. Therefore, the conclusion that the national identification system
is un-American does not depend on this assumption.

(B) Whether the government will soon, or will ever, start curtailing the activities of dissidents is
irrelevant to this argument: that the national identification system is un-American simply be-
cause it restricts the liberties of U.S. citizens. Even if the government does not abuse the power
the national identification system provides, the system could still be considered un-American.

(C) CORRECT. This choice connects the concept of "un-American" policies to restrictions on lib-
erties, essentially defining blanket restrictions on citizens as un-American.
(D) Whether Americans are willing to give up their right to travel freely is irrelevant to this ar-
gument: that the national identification system is un-American simply because it restricts the
liberties of U.S. citizens. Even if Americans were willing to give up their right to move about
without identification, the system could still be considered un-American.

(E) While the author may be inclined to agree that Americans should resist the government
regulation of their lives that the national identification system represents, this argument does
not depend on such an assumption. In fact, the author makes a distinction between the nation-
al identification system and “licenses for purposes…directly related to operating a motor vehi-
cle,” so it is possible that the author considers some government regulation reasonable.

7.

The argument presents the facts of an apparent change in a magazine's cover features since
the new publisher took control. While a gossip columnist hailed the change, newspaper editori-
als disagreed and concluded that the publisher favored profit over reporting. The editorials are
the opponents of the gossip columnist; since their conclusion is about the publisher’s desires,
there must be an assumption connecting the publisher to the covers.

(A) This choice is irrelevant, as it is not connected to the conclusion. The activities of celebrities
have nothing to do with the publisher’s interests.

(B) CORRECT. Since the conclusion concerns the publisher’s desires based on the content of the
magazine covers, the editorials have to assume that the publisher decides who is to be a cover
subject. If not, there is no connection between the covers and the publisher’s interests.

(C) This choice is the opposite of a necessary assumption. For the editorials to conclude that
the publisher prefers profits to reporting, they have to assume that the two are mutually exclu-
sive.

(D) “Some” means “at least one,” so this is not a powerful statement in any direction. Further-
more, even if several such stars were running for political office, it is not at all necessary to as-
sume that to conclude that the publisher was more interested in profits.
(E) This choice is not correct. While it is true that the editorials must assume model and movie
star covers are likely to sell more copies, it does not have to be assumed that such covers will
result in the sale of triple the number of copies, or any other specific number.

8.

The conclusion of the argument is that renewable sources of energy, chiefly solar and wind, will
be less risky for certain utilities than nonrenewable sources, such as oil and gas. The basis for
this claim is that the renewable sources will provide stable, low-cost supplies of energy, where-
as the prices for nonrenewable sources will fluctuate according to availability. We are asked to
find an assumption underlying this argument. In order for this argument to be valid, it must in
fact be true that these renewable sources of energy will provide stable, low-cost supplies.

(A) The utility companies' claim has to do with the supply risk of the new energy sources, not
with how these sources are received by the public.

(B) If no new supplies of traditional energy sources are found, then it is true that perhaps these
nonrenewable supplies will continue to fluctuate in price in a risky manner. However, the argu-
ment does not depend upon any assumption about the future discovery of oil and gas supplies.

(C) CORRECT. If we assume that weather patterns are consistent and predictable, then with the
stated premises, we can conclude that solar and wind power will be less risky than oil and gas.
If, on the other hand, weather patterns are not consistent and predictable, then solar and wind
power are not reliable and thus will not provide "stable energy supplies at low cost." Thus, the
argument's conclusion directly depends on this assumption.

(D) To reach the required conclusion, it is not necessary to assume that the conversion technol-
ogy for new sources is not more expensive than the present technology.

(E) This choice does not directly affect the argument. Whether or not energy produced through
combustion can be made less risky, the new energy sources might still be less risky than the
older sources.
9.

We are given two premises based on survey results: first, vanilla is the best-selling flavor of ice
cream and, second, those who prefer chocolate usually don't order vanilla. The author con-
cludes that vanilla-flavored candy should sell better than chocolate-flavored candy. The author
bases this conclusion on the assumption that it is valid to extend the survey's results beyond ice
cream to include candy.

(A) As a stand-alone, this choice makes common sense but, in the argument, it would under-
mine the author's conclusion. We are asked to find an assumption upon which the author re-
lies, which means the correct assumption should support the author's conclusion.

(B) This choice is either irrelevant at best (the survey does not address children specifically) or
would undermine the author's conclusion, at worst. We are asked to find an assumption upon
which the author relies, which means the correct assumption should support the author's con-
clusion.

(C) Preferences for flavors neither vanilla nor chocolate are outside of the scope of this argu-
ment.

(D) This choice addresses only ice cream preferences; it does not provide any information to tie
ice cream preferences to candy preferences.

(E) CORRECT. This assumption supports the author's conclusion by tying ice cream preferences
directly to candy preferences.

10.
The conclusion of the argument is that the media are wrong in saying that the economy is en-
tering a phase of growth and prosperity. The basis for that claim is that the number of people
filing for bankruptcy has increased every month for the last six months and that bankruptcy
lawyers are busier than they have been in years. In order for this argument to be valid, howev-
er, the author has to assume that the increase in the number of bankruptcies is a result of the
state of the economy and not the result of something unrelated.

(A) This statement does not have to be true for the claim that the media are wrong about the
economy to hold. Even if unemployment rates are useful indicators of growth and prosperity,
the media could still be wrong about the economy (e.g., if there are other indicators that show
problems in other areas).

(B) This does not have to be true for the conclusion to hold. Productivity could be a good
measure of economic growth, but the media could still be wrong about the economy (e.g., if
there are other indicators that show problems in other areas).

(C) CORRECT. This has to be true for the conclusion to hold. If legislation has recently been
passed that makes it easier to obtain bankruptcy, then the evidence cited would be less rele-
vant. The increased number of bankruptcies could have been the result of the easier process
rather than of a poor economy.

(D) This does not have to be true for the conclusion to hold. An increase in the number of
bankruptcy lawyers would not explain the increase in the number of bankruptcy filings.

(E) This does not have to be true for the claim that the media are wrong about the economy to
hold. Even if the media did not often misrepresent the current state of economic affairs, the
argument that the media are wrong might still hold.

12.

The correct answer is B.


The conclusion of the argument is that Michelangelo must have completed the painting be-
tween 1507 and 1509. The basis for that claim is that the painting depicts a coin that did not
exist before 1507 and that it contains a pigment that Michelangelo ceased using in 1509. We
are asked to find an assumption that completes the logic of this argument.

Choice A is incorrect. We do not need to assume that no stocks of the pigment existed after
1509. The argument is concerned only with the year in which Michelangelo stopped using the
pigment.

Choice B is correct. In order to conclude that the painting must have been completed before
1509 on the basis of the pigment, we must assume that he did not begin the painting before
1509 using the old pigment and complete the painting after 1509 with the new pigment.

Choice C is incorrect. The fact that the general public knew of the coin in 1507 is irrelevant to
the conclusion.

Choice D is incorrect. The fact that the panel cannot be tested for age does not relate to either
the coin or the pigment, the two bases for the conclusion.

Choice E is incorrect. Whether Michelangelo's painting style changed during this period does not
relate to either the coin or the pigment.

12.

The conclusion of the argument is that "Company X's fuel costs this year will be significantly
higher than they were last year. Why? Because some of the company's plants switched from oil
to natural gas when the price of gas was lower, and now the price of gas has outstripped the
price of oil. We are asked to find an assumption that is necessary for the argument to work.

(A) Whether Company X has the money to cover its costs does not affect the amount of those
costs.
(B) We do not need to assume that the costs cannot be offset by reducing expenditures in oth-
er areas in order for Company X's costs to be higher.

(C) We do not need to assume that gas will never be cheaper than oil in order for Company X's
costs to be higher.

(D) CORRECT. The author does not take into account the fact that only "some" of the compa-
ny's plants converted to natural gas. Some of the plants, then, still use oil, which is now cheap-
er. So in order to conclude that the company will have to spend more on fuel, the author must
assume that the extra cost of the natural gas for the plants that converted is at least as much
as the cost of the oil for the plants that did not.

(E) We do not need to assume that the price of oil will not suddenly rise in order for the argu-
ment to work.

13.

The advertisement discusses the merits of Avian Oculars, explaining that they are reasonably
priced and contain several popular features. The advertisement concludes, however, by encour-
aging readers to use Avian Oculars in order to see some of the world’s rarest bird species. A
proper assumption must reasonably connect these diverse ideas.

(A) The argument states that Avian Oculars are lightweight, but makes no direct comparison to
traditional binoculars.

(B) Although the advertisement is geared toward bird watchers, nothing indicates that Avian
Oculars should be used only by bird watchers. Even if Avian Oculars were used by individuals
other than bird watchers, no information in the advertisement would be undermined.

(C) CORRECT. The conclusion of the advertisement is that, by using Avian Oculars, the reader
will see some of the world’s rarest bird species on his or her next bird watching trip. In order for
this to be true, the reader would have to have access to these rare bird species. Even the best
bird watching technology would prove useless to view a particular species of bird if the species
were not present.

(D) This choice is true, but it is not an assumption. Remember, an assumption is an unstated
piece of information that ties a conclusion to its premises. This particular answer choice is simp-
ly a stated premise from the second sentence of the argument.

(E) This argument presents one specific product, Avian Oculars, which is specifically designed
with birding enthusiasts in mind. From this information, we cannot assume that birding enthusi-
asts themselves determined the specifications of Avian Oculars, nor can we make any assump-
tions about who determines the specifications of other products.

14.

The basic structure of this argument is fact that “mold is almost always found in places where
there is substantial moisture,” so therefore, to avoid mold and the resultant mold poisoning,
then people should take steps to prevent wet areas. This argument assumes that wet areas oc-
cur first, causing mold to grow. Conversely, this assumption requires that the mold growth itself
does not occur first, creating wet areas as a result.

(A) CORRECT. The argument depends on the assumption that the reason mold and wetness are
observed together is that wet areas cause mold growth. If the reverse causation (mold causes
wetness) were true, then keeping all plumbing in good condition to prevent leakage would do
little to prevent the growth of mold. This choice eliminates the alternate causation.

(B) If most homeowners know enough about plumbing to determine whether theirs is in good
condition, then the recommendation made in this argument would be more useful. However,
this is not an assumption on which the argument depends.

(C) Even if mold could grow in dry areas, the fact that mold is almost always found in wet areas
is still valid. This is the fact upon which the argument is based, so the argument does not de-
pend on the unnecessarily absolute assertion that mold cannot grow in dry areas.
(D) Even if some varieties of mold are harmless, the conclusion of this argument, that “one
should make sure to keep all internal plumbing in good condition to prevent leakage” and mini-
mize mold growth, could still be valid. Therefore, this argument does not depend on the unnec-
essarily absolute assertion that no varieties of mold are harmless.

(E) Whether mold spores can be filtered from the air may be relevant to a conclusion about the
health effects of mold in the home, but it is not directly relevant to this conclusion, that “one
should make sure to keep all internal plumbing in good condition to prevent leakage” and mini-
mize mold growth.

15.

The conclusion of the argument is that the majority of American citizens believe in the death
penalty while the majority of Filipino citizens do not. This conclusion is based solely on the fact
that the death penalty is legal in the United States while it is now illegal in the Philippines. The
argument assumes that there is a link between the legal status of capital punishment and the
beliefs of the majority of citizens.

(A) While the number of murders per year might influence beliefs about capital punishment,
there is certainly no necessary correlation. Moreover, the difference in the number of murders
per year in the United States and the Philippines has little relevance without knowing the rela-
tive populations of the two countries.

(B) CORRECT. It is possible that the legal status of capital punishment in the United States and
the Philippines does NOT align with how the majority of citizens in those countries view the
death penalty. For example, it may be that the governing bodies of one or both countries are
out of touch with the views of the populace. The argument assumes that this is NOT the case.

(C) Even if there were strong voices opposing the death penalty in the United States, the argu-
ment might still hold. The argument is based on the views of the majority of citizens in the
United States, not on all US citizens.

(D) The argument addresses whether or not citizens of the United States and the Philippines
believe in the death penalty. The reasons behind those beliefs, as presented in this choice, are
irrelevant to the logic of the argument.
(E) The argument is centered on whether or not citizens of the United States and the Philip-
pines believe in the death penalty. While the legal standard used to sentence criminals to the
death penalty might impact why individuals hold certain beliefs, it has no impact on the logic of
the argument.

16.

Although the premises of this argument suggest only a correlation between smoking and anxie-
ty or nervousness, the argument has a causal conclusion: it concludes that smoking causes in-
dividuals to be anxious and nervous (i.e., that A causes B). Any assumption in a causal argu-
ment must support the causal “direction” of the conclusion, that A causes B as opposed to some
other explanation. Often, assumptions support a causal conclusion either by eliminating an al-
ternate cause for the conclusion (that C did not cause B) or by demonstrating that the causa-
tion, if one exists, is in the proper direction (that B did not cause A).

(A) The argument concludes that smoking causes anxiety and nervousness. Whether these
maladies lead to more serious health problems is not relevant to the conclusion.

(B) CORRECT. For smoking to be the cause of anxiety and nervousness (i.e., that A caused B)
it must be true that these individuals were not more likely to be anxious and nervous before
they started smoking. If smokers had these preconditions, which contributed to their decision
to begin smoking (i.e., that B caused A), our conclusion – that smoking causes these maladies –
would be incorrect.

(C) The argument concludes that smoking causes anxiety and nervousness. The number of sur-
vey respondents is not relevant to the conclusion.

(D) The argument concludes that smoking causes anxiety and nervousness. The awareness of
the health problems related to smoking is not relevant to the conclusion.

(E) The argument is not based on the immediate impact that smoking has on anxiety and nerv-
ousness. Moreover, the argument never compares some smokers to other smokers.
17.

The argument concludes that the stock of the firm will experience rapid growth. The basis for
this claim is that the firm has shown strong historical performance that is likely to continue in
the future. The stock will appreciate dramatically in the future as a result only if it has not al-
ready appreciated in anticipation of the company's expected growth.

(A) The argument focused on the potential for stock appreciation rather than company weight
in the industry. A company with a large market share may well experience poor stock perfor-
mance, while a company with a small market share may continue to grow and increase in val-
ue.

(B) Since the conclusion of the argument is made regarding the future outlook, it is not neces-
sary to assume that the company had been growing, or had even existed, prior to the past 5
years. A new firm that has been in existence for only 5 years may well present an excellent in-
vestment opportunity.

(C) This statement is explicitly stated in the argument and therefore does not have to be as-
sumed.

(D) CORRECT. If this assumption were not true, i.e. if the current stock price already reflects
future growth prospects, then the premise that the company will experience high growth is cer-
tainly insufficient to warrant future stock price appreciation, since all of this growth would al-
ready be reflected in the current price. It is necessary to assume that the current price of Com-
pany X stock does not yet reflect the promising growth prospects of the firm, allowing the pos-
sibility that the stock price will rise further.

(E) Note that the argument makes a claim about the absolute return of stock X rather than its
return relative to the industry. Therefore, to justify the rapid growth in the stock price, it is not
necessary to assume that the company will outperform its competitors. For example, if the in-
dustry itself is growing very rapidly, other companies in the industry can experience just as rap-
id appreciation in stock prices.
18.

Antoine is alarmed that the number of children on antipsychotic medication has increased by 73
percent. Lucy begins her reply with “but,” indicating that she is about to counter either An-
toine’s facts or his alarm; she accepts his facts but addresses his alarm. If the number of chil-
dren taking antipsychotic medication is still within the normal range, the rate at which the total
number has increased is not cause for alarm. Lucy uses information about adult use of such
drugs to imply that the lower rate of antipsychotic drug use in children must also be normal.

(A) Lucy’s argument is about a normal level of antipsychotic drug use; how rarely or frequently
that level is exceeded is outside the scope of her argument.

(B) Lucy uses the percentage of adults taking antipsychotic medication to illustrate normal lev-
els of the use of such drugs. It happens that the percentage of children taking such medication
last year was lower than the percentage of adults, but her argument does not require the as-
sumption that that will always be the case.

(C) CORRECT. If there is no difference between children and adults on the matter of antipsy-
chotic drug use, then Lucy can legitimately use information about adult use of such drugs to
imply that the lower rate of antipsychotic drug use by children must also be normal. On the
other hand, if this assumption were not valid – for example, if children responded differently to
the drugs, or if the rate of the drug use by adults is considered too high for children – then Lu-
cy’s statement would not be enough to address Antoine’s alarm.

(D) Lucy’s argument is not based on the figure Antoine cites and does not assume its accuracy
or inaccuracy. Rather, her argument uses the relative adult and child rates of antipsychotic drug
use to point out that Antoine’s statistic is not inconsistent with a normal rate of such use in chil-
dren.

(E) The fear of random violence by adolescents is not part of Lucy’s argument; this statement is
irrelevant.

19.
The question asks for an assumption made by the reader. The reader’s conclusion is that aca-
demic rigor is in decline, based on the percentage of colleges granting a majority of their de-
grees in the liberal arts mentioned in the article. To draw this conclusion, the reader must as-
sume that degrees not in the liberal arts were not as academically rigorous.

(A) It is not necessary for the reader to assume that the percentage will continue to drop. The
reader's conclusion concerns the present. Assumptions must be both unstated and necessary.

(B) This extreme statement is not a necessary assumption. The reader does not have to as-
sume that all colleges should do so; the conclusion only relies on an assumption that 5.5% is
too low.

(C) CORRECT. To conclude that the low percentage of colleges granting the majority of their
degrees in the liberal arts indicates a decline in academic rigor, the reader must assume that
other degree programs required less academic rigor. If not, this evidence would not indicate a
decline in academic rigor.

(D) This is not a necessary assumption. The relative importance of academic rigor is irrelevant
to the reader’s claim. That claim only asserts that academic rigor, in isolation, is in decline. The
claim has nothing to do with its importance relative to other attributes.

(E) It is not necessary to assume anything specific about the schools that do not grant a majori-
ty of their degrees in the liberal arts, as they are not the subject of the evidence or the conclu-
sion. The reader feels that the low percentage mentioned is evidence enough; it is not neces-
sary to assume any arbitrary level below the 50% of degrees standard that the article and the
reader use.

20.

The doctor concludes that federal legislation prohibiting the sale of video games to minors
would help reduce the incidence of carpal tunnel syndrome. This conclusion hinges on the as-
sumption that the only way for adolescents to access video games is to purchase the games
themeslves.
(A) Majority consensus in the legislature has no bearing on whether the recommended legisla-
tion would actually help to curb carpal tunnel syndrome.

(B) This argument states that "adolescents who play video games on a regular basis are three
times as likely to develop carpal tunnel syndrome." Thus, the argument directly indicates that
carpal tunnel syndrome does not affect all adolescents who play video games. Rather than an
assumption, this answer choice is simply an inference drawn from the text.

(C) The fact that adolescents can develop carpal tunnel syndrome by means other than playing
video games has no bearing on whether the recommended legislation would help to curb carpal
tunnel syndrome.

(D) CORRECT. In order for the doctor's recommended legislation to reduce the incidence of
carpal tunnel syndrome among adolescents, the prohibition from the purchase of video games
must result in the actual possession of fewer video games. Thus, it must be assumed that par-
ents will not simply purchase video games for their children.

(E) The fact that video games can benefit adolescents in other ways has no bearing on whether
the recommended legislation would help to curb carpal tunnel syndrome.

21.

The management concludes that a 10% increase in gasoline prices will result in a 10% increase
in revenues from the sale of gasoline. In order to reach this conclusion, we need to assume that
the amount of gasoline sold will not drop despite the higher prices.

(A) CORRECT. This assumption is critical to justify the projection that a 10% increase in gas
prices will result in a 10% increase in revenues from gasoline sales. Note that if this assumption
does not hold, the management’s projection will collapse. For example, if consumers switch to
public transportation or simply start to drive less in response to the higher prices, the revenues
of the company will not increase by the same amount as the increase in the sales price. In fact,
if the decline in gasoline consumption is substantial (e.g. 20%) the company will experience
lower rather than higher revenues.
(B) The issue of profits is irrelevant to the management’s conclusion about revenues from the
sale of gasoline.

(C) Since the management’s projection concerns only the sales of gasoline, revenues of other
business lines are beyond the scope of the argument.

(D) Since the management’s conclusion concentrates on revenues, the issue of costs is beyond
the scope of the argument.

(E) It is not necessary to assume that the supply of gasoline will decline, since the price in-
crease can be driven by a variety of other factors, such as production costs, market environ-
ment, and others.

22.

The conclusion of the argument is contained in the last sentence, that "the methods that the
prospector had used to determine the size of the oil deposit must have been inaccurate." The
evidence provided is that the prospector reported a large oil deposit that was later determined
to be much smaller in size. We are asked to find an unstated assumption that makes the con-
clusion valid based upon this evidence. In order to do this, we need to assume that there is not
another reason why the prospector might have reported a larger oil deposit than actually exist-
ed.

(A) It is not necessary to the conclusion that a third party affirmed the company's determina-
tion that the oil deposit turned out to be small. The conclusion accepts that the oil deposit was
indeed smaller than indicated by the prospector, and focuses on the cause of the discrepancy
as opposed to the discrepancy itself.

(B) CORRECT. The argument concludes that the prospector's methods resulted in inaccurate
measurements of the size of the oil deposit. This assumes that the prospector did not simply
misreport or misrepresent the measurements, presumably for personal gain. This answer
choice addresses the most plausible alternative explanation to the one given, and is necessary
for the conclusion to stand based upon the evidence presented.
(C) The commercial feasibility or profitability of the oil deposit is not integral to the argument,
or its conclusion. This statement is not an assumption that would support the conclusion.

(D) Whether or not the prospector utilized the same methods in regards to measuring the oil
deposits in other locations is not relevant to the argument, or the conclusion.

(E) The fact that the company had a long operating history and experience in drilling oil wells
is not relevant, in that the company's measurements of the size of the oil field are accepted as
given in the argument.

The correct answer is B.

23.

This argument concludes that the city needs educational leadership that can solve problems,
not create them. It illustrates this claim by discussing the prohibition on cell phones. This prohi-
bition is given as an example of the leadership creating problems where none exist. The neces-
sary assumption is one that bridges the logic gap between the premise and the conclusion.

(A) CORRECT. In order to use the issue of students having cells phones as an example of how
the educational leadership creates problems where none exist, the author must assume that
students having cell phones is not an important problem.

(B) The argument and its conclusion are not about students’ needs. They concern the quality of
educational leadership. Thus, this choice is irrelevant; it is not necessary for the argument to
assume anything about students’ needs.

(C) This choice is irrelevant; the argument does not concern faculty and staff cell phones, and
thus no assumption about them is necessary. Assumptions must be both unstated and neces-
sary to the conclusion.
(D) The argument and its conclusion are not about students’ needs. They concern the quality of
educational leadership. Thus, this choice is irrelevant; it is not necessary for the argument to
assume anything about students’ needs.

(E) The argument does not rank the various attributes of good educational leadership. It only
discusses one quality. Thus, no assumption about the relative importance of attributes is neces-
sary, and this choice is irrelevant.

24.

The argument proposes that directing chiropractic treatment toward adults with severe back
problems is the best way to minimize the proportion of the population that suffers from back
pain. The argument assumes a high degree of overlap between those adults who suffer from
severe back problems and all people who suffer from back pain, where the former group may
comprise only a subset of the latter group. It also assumes that there is not another recom-
mendation that could better accomplish the goal of minimizing back pain for a larger segment
of the population than treating just those adults with severe back problems.

(A) The argument does not depend upon whether or not chiropractic treatment can or cannot
be used in conjunction with other medical treatments. The fact that people could benefit from
other forms of treatment aside from chiropractic care weakens the argument to a slight degree,
and does not act as a supportive assumption.

(B) The relative degree of insurance coverage of chiropractic care compared with other medical
treatments is not vital to the argument. Also, that insurance carriers cover chiropractic care to a
lesser degree weakens the argument marginally, and does not act as a supportive assumption.

(C) CORRECT. This statement rules out the possibility that chiropractic care or other medical
treatments could effectively prevent or lessen back pain among those people who have not yet
developed severe back problems.

(D) That chiropractic treatment is more effective over time is irrelevant to the argument.
(E) The economic impact of severe back pain and other problems is not addressed in the argu-
ment. Instead, addressing back pain is treated as an intrinsic goal, with no mention of economic
externalities.

25.

The argument presents the facts of an apparent change in a magazine's cover features since
the new publisher took control. While a gossip columnist hailed the change, newspaper editori-
als disagreed and concluded that the publisher favored profit over reporting. The editorials are
the opponents of the gossip columnist; since their conclusion is about the publisher’s desires,
there must be an assumption connecting the publisher to the covers.

(A) This choice is irrelevant, as it is not connected to the conclusion. The activities of celebrities
have nothing to do with the publisher’s interests.

(B) CORRECT. Since the conclusion concerns the publisher’s desires based on the content of the
magazine covers, the editorials have to assume that the publisher decides who is to be a cover
subject. If not, there is no connection between the covers and the publisher’s interests.

(C) This choice is the opposite of a necessary assumption. For the editorials to conclude that
the publisher prefers profits to reporting, they have to assume that the two are mutually exclu-
sive.

(D) “Some” means “at least one,” so this is not a powerful statement in any direction. Further-
more, even if several such stars were running for political office, it is not at all necessary to as-
sume that to conclude that the publisher was more interested in profits.

(E) This choice is not correct. While it is true that the editorials must assume model and movie
star covers are likely to sell more copies, it does not have to be assumed that such covers will
result in the sale of triple the number of copies, or any other specific number.

26.
The argument concludes that for-profit colleges enroll a greater proportion of financially disad-
vantaged students than do non-profit colleges. This conclusion is based on the fact that stu-
dents at for-profit colleges draw a disproportionate share of federal and state financial aid. The
argument assumes a link between the proportion of aid received and the proportion of finan-
cially disadvantaged students enrolled. In so doing, it assumes that there are not other possible
reasons for the disproportionate aid distribution.

(A) The conclusion makes a claim about the differences between for-profit and non-profit col-
leges. Differences among non-profit colleges – such as public vs. private – are irrelevant to the
argument.

(B) CORRECT. One alternative reason that might explain the disproportionate aid distribution is
that for-profit colleges engaged in fraudulent practices to obtain unneeded financial assistance
for their students. If this were true, then much of the aid was distributed based not on the ac-
tual financial situation of the students but on the ability of colleges to defraud federal and state
governments. This answer choice asserts that this was NOT in fact the case, thereby eliminating
this alternative explanation and highlighting a key assumption upon which the argument rests.

(C) The argument's claim is centered on proportions. The actual number of students receiving
aid at for-profit vs. non-profit colleges is irrelevant to the conclusion.

(D) The relative educational quality of for-profit vs. non-profit colleges lies outside the scope of
the argument, which is focused solely on differences in financial aid distribution.

(E) The issue addressed by the argument is the amount of financial aid distributed to students
at two types of institutions. Whether students successfully repay their loans after college is im-
material to the claim made in the argument.

27.

Research indicates that there is a connection between being married and being happy and
healthy. Media commentators have concluded that marriage causes happiness and health.
However, one could reasonably conclude from the research that the cause and effect are the
reverse: being happy and healthy makes a person more likely to get married.
(A) The research compared married people to unmarried people. Neither the researchers nor
the media commentators made any distinction between newlyweds and those who had been
married a long time, so this assumption is not necessary.

(B) The type of wedding is outside the scope of this argument. The research compared married
people to unmarried people, but made no distinction based upon the type of wedding. Thus,
this assumption is unnecessary.

(C) At first, this statement may seem necessary—after all, if the commentators conclude that
marriage causes happiness, a lack of depression in married people would certainly support that
conclusion. However, the statement is too extreme. One depressed married person does not
invalidate the research indicating that, on average, married people are healthier and happier
than non-married people.

(D) CORRECT. This statement eliminates the alternative interpretation of the research find-
ings—that being happy and healthy makes a person more likely to get married.

(E) The research compared married people to unmarried people. Neither the researchers nor
the media commentators made any distinction between harmonious marriages and combative
marriages, so this assumption is not necessary.

28.

The text tells us only that Country X imposes heavy tariffs on imported goods and that Compa-
ny Y believes it can increase long-term profits by opening a factory in Country X so it can avoid
having to import its goods into Country X. We are asked to select an answer choice that is an
assumption required for Company Y's belief to be valid.

(A) While this is a tempting answer, it is not necessary to assume that Company Y will be able
to obtain all necessary permits. The text does not indicate whether Company Y will actually be
able to implement the plan, only that the plan could increase profits if implemented.
(B) We are given no information about Company Y's activities in other countries.

(C) CORRECT. In order for Company Y to conclude that it can increase long-term profits by
opening a factory in Country X, it must believe that a sustainable market exists for its products
in that country. Otherwise, the new factory would not generate revenue and the company could
not recoup the cost of the new factory.

(D) We are given no information about tariffs in Company Y's home country.

(E) We need not assume that labor costs are lower in Country X. It could be that labor costs in
Country X are higher than those in Company Y's home country but the increased cost of labor is
still less than the tariffs. This would result in a net savings for Company Y in Country X.

29.

The text tells us only that Country X imposes heavy tariffs on imported goods and that Compa-
ny Y believes it can increase long-term profits by opening a factory in Country X so it can avoid
having to import its goods into Country X. We are asked to select an answer choice that is an
assumption required for Company Y's belief to be valid.

(A) While this is a tempting answer, it is not necessary to assume that Company Y will be able
to obtain all necessary permits. The text does not indicate whether Company Y will actually be
able to implement the plan, only that the plan could increase profits if implemented.

(B) We are given no information about Company Y's activities in other countries.

(C) CORRECT. In order for Company Y to conclude that it can increase long-term profits by
opening a factory in Country X, it must believe that a sustainable market exists for its products
in that country. Otherwise, the new factory would not generate revenue and the company could
not recoup the cost of the new factory.

(D) We are given no information about tariffs in Company Y's home country.

(E) We need not assume that labor costs are lower in Country X. It could be that labor costs in
Country X are higher than those in Company Y's home country but the increased cost of labor is
still less than the tariffs. This would result in a net savings for Company Y in Country X.

30.

The conclusion of the argument is that companies should allow other manufacturers to license
patented technology. The basis for that claim is that not doing so keeps prices high and harms
the consumer. We're asked what the argument assumes ("presupposes") in drawing its conclu-
sion. The correct answer will fill the logic gap between the idea that keeping prices high harms
the consumer and that companies should allow other manufacturers to license patented tech-
nology. The conclusion is based on the assumption that companies have an obligation of some
kind to do what's best for the consumer.

(A) This does not address the moral obligation to the consumers (i.e. “should”) of the compa-
nies who produced the patented technology, the main point of the conclusion. Furthremore,
even if companies could find legal ways to produce similar technologies, the patented technolo-
gy could still command exorbitant prices, thereby harming the consumer.

(B) CORRECT. The conclusion only makes sense if companies have an obligation to act in the
best interest of the customer, as this choice states.

(C) This generally follows along with the author's claim, but we are not required to assume this
in order to reach the conclusion that companies who are granted patents are obligated to look
out for the best interests of their customers.
(D) This addresses a tangential issue of whether or not consumers could notice the difference
between a new patented technology and a possible imitation. This does not address the core
issue of the obligation to the consumer.

(E) This does not address the obligation of the companies toward the consumers, or indeed the
companies at all.

31.

The conclusion of the argument is that one need not worry about ingesting pesticides when
purchasing produce from farms that use only organic pesticides. The basis for that claim is the
fact that organic pesticides leave the surface of produce within a few hours of spraying. In or-
der for this argument to be valid, we must assume that the organic pesticides do not harm the
produce in any lasting way.

(A) This is unrelated to the argument since the conclusion speaks about not having to worry
about ingesting produce on which only organic pesticides were known to be used.

(B) If anything, this statement runs counter to what the argument is saying. If produce that has
been sprayed with organic pesticide reaches the final consumer within hours after it is picked, it
is possible that the consumer does need be concerned about the pesticides.

(C) The conclusion of the argument is already limited to those farms which use "only organic
pesticides."

(D) CORRECT. If a pesticide is capable of penetrating the skin of a fruit or vegetable then, while
the organic pesticide will dissipate from the surface of the fruit in a few hours, it may remain
inside the fruit. The author of this argument assumes that the pesticides cannot penetrate the
skin.

(E) The issue of cost is unrelated to the argument's conclusion about pesticide residues.
32.

The conclusion of the argument is that the prediction of decreased consumer spending has not
come to pass. The evidence for this is that there has been no corresponding increase in the
amount of money set aside in savings accounts by the general public. This question asks us to
find an assumption on which this argument is based. The author assumes that the mere fact
that people generally have not been adding more money to their savings accounts means they
have not cut down on their discretionary purchases (i.e., non-essential items). In order for this
to be valid, we must assume that a savings account is the only mechanism by which someone
would save any extra money when reducing discretionary spending.

(A) The state of the economy in the last few months has no bearing on the claim that a lack of
extra money deposited in consumer savings accounts is evidence that consumer spending has
not decreased.

(B) CORRECT. If there was an alternate explanation for the lack of increase in savings accounts,
the claim that the decrease in spending has not taken place would be greatly weakened. This
assumption guarantees us that at least one other possible explanation for the lack of increase in
savings (i.e. an increase in stocks, certificates of deposit, or other savings vehicles) is NOT true.

(C) Assuming that salaries decreased would actually weaken this argument. If salaries de-
creased, that may explain the decrease in the amount of money being put aside in savings
without necessarily implying a decrease in spending. People could be making less, spending the
same, and therefore saving less.

(D) The lack of growth in business sectors has nothing to do with what consumers are doing
with their money during the same time period.

(E) The argument says the prediction was made "even [for those whose] jobs were secure."
People who find other employment, therefore, would still fall under the economists' prediction
and would, if the prediction were true, be expected to cut discretionary spending and save
more money.
33.

The conclusion is that nurses should examine patients to determine which deserve to be seen
first by the doctors. The basis for this claim is that hospitals lack adequate numbers of physi-
cians.

(A) The idea of having nurses make the initial examination does not depend on increasing the
medical staff.

(B) The main premise for the conclusion was that patients ended up waiting due to an under-
supply of doctors. There weren't enough doctors to perform the initial examination. If the doc-
tors perform the initial examinations there will be no time saved.

(C) The conclusions rests on whether or not the nurses would be able to perform the examina-
tions, not on what the result of them doing the examinations would be.

(D) The hospitals don't need to be fully staffed with nurses for the nurses to perform the initial
examination.

(E) CORRECT. This argument is valid only if we assume that nurses are competent to determine
which patients merit immediate treatment.

The correct answer is E.

34.

The correct answer is B. The conclusion of the argument is that "the scientists must have dis-
covered the butterfly at night." Why? Because the butterfly's color matches the green of the
foliage, and the butterfly is active only at night. In order for this argument to work, one has to
assume that there is no way for the scientists to detect the butterfly during the day, despite its
camouflage. If there is a way for the scientists to detect the butterfly during the day, the con-
clusion is no longer logical.

You might also like